LSAT and Law School Admissions Forum

Get expert LSAT preparation and law school admissions advice from PowerScore Test Preparation.

 Administrator
PowerScore Staff
  • PowerScore Staff
  • Posts: 8917
  • Joined: Feb 02, 2011
|
#33609
Complete Question Explanation
(See the complete passage discussion here: lsat/viewtopic.php?t=13780)

The correct answer choice is (A)

This is a relatively challenging question, in which we are asked to strengthen the reasoning by which Estabrook justifies his choice of artistic techniques. From the fourth paragraph, we know that he embraces their imperfections in order to achieve certain aesthetic effects. The correct answer choice must defend this view.

Answer choice (A): This is the correct answer choice. If advanced photographic techniques cannot faithfully reproduce the desired imperfections in nineteenth-century prints, it would be reasonable for Estabrook to resort to older processes in order to make more faithful reproductions. This answer choice helps justify Estabrook’s choice of strategy by eliminating an alternative strategy for achieving his artistic objectives.

Answer choice (B): This is the Opposite answer. If the feelings evoked by a work of art were irrelevant to its artistic value, there would be little reason to produce prints that evoke sentiments such as nostalgia. This answer choice would undermine Estabrook’s artistic objectives.

Answer choice (C): The ability of photographs to record subjects accurately has no bearing on Estabrook’s artistic objectives. To convey the experience of nostalgia, Estabrook embraces accident and idiosyncrasy, not accuracy. This answer choice is incorrect.

Answer choice (D): This is another Opposite answer, because it undermines Estabrook’s decision to create fake antique photographs. If such prints were of little artistic value, his choice of strategy would be seriously compromised.

Answer choice (E): The ability of modern techniques to produce perfect photographs has no bearing on Estabrook’s decision to use outdated techniques, because his goal is to create artistic prints that are imperfect. This answer choice is incorrect.
 SherryZ
  • Posts: 124
  • Joined: Oct 06, 2013
|
#11896
June 2013 LSA Sec 3 RC, Q14:

Could you explain why A is correct? Where can we find "Betray the artifice involved" in the passage??

Thank you very much!

---Sherry
User avatar
 KelseyWoods
PowerScore Staff
  • PowerScore Staff
  • Posts: 1079
  • Joined: Jun 26, 2013
|
#11906
Hi Sherry!

To answer this question, we first need to figure out how Eastbrook justifies his choice of tintype photography. According to the passage (second half of paragraph 4), he uses tintype photography so that his photographs will have imperfections that "heighten the sense of nostalgia." Answer choice (A) strengthens his reasoning because it says that if he used modern photographic techniques to intentionally create imperfections, the viewers would be able to tell that the photograph had been manipulated (the prints would "betray the artifice") and so they would not get that heightened sense of nostalgia. Therefore, to give that heightened sense of nostalgia, he needs to use the old methods.

"Betray the artifice" is not involved in the passage. It's merely saying that viewers would be able to tell if you used modern technology to manipulate photographs to look old rather than using authentic tintypes.

Hope this helps!

Best,
Kelsey

Get the most out of your LSAT Prep Plus subscription.

Analyze and track your performance with our Testing and Analytics Package.